determine whether the following polygons are similar. if yes, type 'yes' in the similar box and type in the similarity statement and scale factor. if no, type 'none'​

Answers

Answer 1

Similar: yes

Similarity Statement: AB ~ TQ , BC ~ QR

Scale Factor: 4/3


Related Questions

Find the lateral surface area of this
cylinder. Round to the nearest tenth.
r = 3 cm
3 cm

Answers

Answer:

22/7×9×3=22×9×3=594÷7=84.85

Answer:

56.5 For Acellus Users

Step-by-step explanation:

Step 1. 2 x 3.14(pi) x radius(3) x height(3)

Step 2. Once added up should equal 56.5

Remember 2 x 3.14 x 3 x 3 = 56.5

Part 1: - What is the end-of-year wealth if Jane Christine receives a stated annual interest rate of 24% compounded monthly on a $1 investment? - Harry DeAngelo is investing $5,000 at a nominal interest rate of 12%, compounded quarterly, for five years. What is his wealth at the end of five years? - Linda Defond invested $1,000 at a continuously compounded rate of 10% for two year. What is the value of her investment at the end of two years?

Answers

Answer:

$1.27

$9030.56

$2,210.34

Step-by-step explanation:

The formula for calculating future value:

FV = P (1 + r/n)^nm

FV = Future value  

P = Present value  

R = interest rate  

m = number of compounding

N = number of years  

1. 1( 1 + 0.24 /12)^12 = $1.27

2. 5000 ( 1 + 0.12 /4)^( 5 x 4) = $9030.56

the formula for calculating future value when there is continuous compounding is : A x e^r x N

A= amount e = 2.7182818 N = number of years r = interest rate

3. 1000 x e^0.1 x 2 = $2,210.34

At Fireflies Kids Camp, 4 out of the 10 kids in the yellow cabin are taking swimming lessons. In the blue cabin, 5 out of the 15 kids are taking swimming lessons. Do both cabins have the same ratio of swimmers to total kids?

Answers

Answer:

Both cabins do not have the same ratio of swimmers to total kids.

Step-by-step explanation:

The ratio of swimmers to total kids in the yellow cabin is:

4 : 10

The ratio of swimmers to total kids in the blue cabin is:

5 : 15

4 : 10 and 5 : 15 are not equivalent ratios because:

LCM of 10 and 15 is 30

Multiply each term in a ratio by a number so that the second term is 30:

4 x 3 = 12

10 x 3 = 30

4 : 10 = 12 : 30

5 x 2 = 10

15 x 2 = 30

5 : 15 = 10 : 30

12 : 30 and 10 : 30 are NOT the same ratios.

Hope this helps!

You are filling a bookcase with books. The bookcase is 3 feet wide. Your hardcover books are 1 1/2 inches wide and your paperback books are 3⁄4 inches wide. If you have already placed 8 hardcover books on the shelf, what is the maximum number of paperback books you can also fit on the shelf?

Answers

Answer:

32 paperback books

Step-by-step explanation:

Given that :

Width of hardcover books = 1 1/2 inches

Width of paperback books = 3/4 inches

Number of hardcover books already on shelf = 8

Total width of shelf = 3 feets

1 foot = 12 inches

Hence, total shelf width in inches = (12 * 3) = 36 inches

Total width of hardcover books = (8 * 1 1/2) = 12 inches

Total shelf width left = (36 - 12) inches = 24 inches

Maximum number of paperback books :

Total shelf width left / paperback width

24 inches / (3/4)inches = 32 paperback books

solve x
solve x
solve x

Answers

Answer:

x = 25

Step-by-step explanation:

When you combine both angles, you can get a supplementary angle which means that when they are both added, it should add up to be 180 degrees. With that being said, we can create an equation and solve for x.

5x - 5 + 2x + 10 = 180

~Combine like terms

7x + 5 = 180

~Subtract 5 to both sides

7x = 175

~Divide 7 to both sides

x = 25

Best of Luck!

Answer: x = 25

Step-by-Step Explanation:

We are given a line, hence it is a straight angle being 180°

Therefore,

=> 5x - 5 + 2x + 10 = 180

= 5x - 5 + 2x = 180 - 10 = 170

= 5x + 2x = 170 + 5 = 175

= 7x = 175

=> x = 175/7 = 25

Therefore, x = 25

Additionally, finding the values of each angle :-

=> 5x - 5

= 5(25) - 5

= 125 - 5

=> 120°

=> 2x + 10

= 2(25) + 10

= 50 + 10

=> 60°

Therefore, one angle is 120° and the other is 60°

Please help with question thank you

Answers

Answer:

The answer is 3x=50-10y

this is the answer 3x = 50 - 10y

the least common multiple of ½,⅓,⅘ and ³/¹⁰ is a)90 b)50 c)30 d)15 e)10​

Answers

Yhhshussuudhddhhdhdhdhdhfidijenrnrr

Answer:

Step-by-step explanation:

Please hurry I will mark you brainliest

Answers

Answer:

12500

Step-by-step explanation:

11830 - 10464 = 1366 (difference 1995 to 96)

10464 - 8958 = 1506 (94 to 95)

8958 - 8135 = 823

8135 - 7537 = 598

7537 - 6642 = 895

6642 - 5942 = 700

5942 - 3579 = 2363 (5 years)

so, we have the annual differences over 11 years.

the expectation for the following year would be based on the mean value of growth rate and not on the mean value of the absolute numbers, as we know the number of transactions for 1997 will be higher than for 1996.

so, the mean value of growth over these 11 years is 750.

so, based on this we would estimate for 1997

11830 + 750 = 12580 (12500 being the closest answer option).

but if the business environment has changed over the most recent years, we should not bring in earlier years into that calculation (or at least not with the same weight).

we could argue that the last 2 years had a totally different behavior than the years before.

so, just creating a mean value of the last 2 years' changes would be

(1366 + 1506) / 2 = 2872 / 2 = 1436.

now, or prediction would be

11830 + 1436 = 13266, which would be closer to the 13500 answer option.

Whats the correct answer?

Answers

Answer:

Actually the answer is "A"

this is a very strange way to present a problem...

this issue her is that [tex](x^{a}) ^{b} = x^{a*b}[/tex]

so you need 1/3 * 3 in the answer... none of them have 1/3 * 3

BUT !!!!! 1/3 + 1/3 + 1/3  is the same as  1/3 * 3 So "A" is the solution

Step-by-step explanation:

a parabola has x-intercepts at x = 1/2 and x=5. what is the equation of the parabola

Answers

Answer:

 y = 2x²-11x +5    

Step-by-step explanation:

We know that a polynomial of degree n with roots (x-intercepts) {x₁, x₂, ..., xₙ} and a leading coefficient A can be written as:

p(x) = A×(x - x₁)×...×(x - x₂)

Here we will find:

P(x) = (x - 1/2)×(x - 5)

Let's see how we found that:

Here we know that is a parabola, so n = 2, and the x-intercepts are:

x₁ = 1/2

x₂ = 5

And we do not know the leading coefficient, so we can assume  A = 1.

Then the polynomial is just:

P(x) = (x - 1/2)×(x - 5)

The graph of this polynomial can be seen below.

If you want to learn more, you can read:

https://brainly.com/question/24371640

Marcus plans to spend $72 at a local clothing store that sells t-shirts for $8 each and jeans for $12 each, sales tax included. This scenario is modeled by 8x + 12y = 72, where X represents the number of t-shirts purchased and y represents the number of jeans purchased.
What point on the graph represents Marcus' purchase if it consisted of only t-shirts?

Option
A. The x intercept (9,0)
B. The y intercept (0,6)
C. The x intercept (6,0)
D. The y intercept of (0,9)

Answers

Answer:

a. the x intercept (9,0)

Help please
Which points lie in the third quadrants? check all that apply.
A. Point F
B. Point B
C. Point C
D. Point D
E. Point E

Answers

Answer:

A. Point F

Step-by-step explanation:

Quadrant I is the top right, Quadrant II is the top left, Quadrant III is the bottom left, and Quadrant IIII is the bottom right. SO basically, the order of quadrants is counterclockwise.

If we look at the bottom left (Quadrant III), we see there is only one point there; point F.

Hope this helps!

If there is something wrong, please let me know.

Find x round your answer to the nearest integer.

Answers

Answer:

C

Step-by-step explanation:

Note that x is opposite to the given angle and we are also given the hypotenuse.

Since we have an angle and the side opposite to it and the hypotenuse, we can use the sine ratio. Recall that:

[tex]\displaystyle \sin \theta =\frac{\text{opposite}}{\text{hypotenuse}}[/tex]

The opposite side is x, the hypotenuse is 15, and the angle is 53. Substitute:

[tex]\displaystyle \sin 53^\circ = \frac{x}{15}[/tex]

Solve for x:

[tex]x=15\sin 53^\circ[/tex]

Use a calculator (make sure you're in Degrees Mode!). Hence:

[tex]x=11.9795...\approx 12[/tex]

Our answer is C.

Answer:

C. 12

Step-by-step explanation:

Pythagoras Theorem

Please Help!

Use the Parabola tool to graph the quadratic function. f(x)=3x^2−6x+5 Graph the parabola by first plotting its vertex and then plotting a second point on the parabola.

Answers

you didn't tell wich program to use and did not include any pictures.

anyways, I graphed the function with desmos (free to use as app and on website)

so that u can see what the graph should look like.

variance based on the zoom is okay of course.

Find the value of x and HE in the triangle

Answers

Answer:

EH = HF by the perpendicular bisector theorem

equation:

3x=x+4

or, 3x-x=4

or, 2x=4

or, x=2

so, HE = 6

Answered by GAUTHMATH

how many candy bars did team sell if they collected $1,00.00

plz help
ASAP

Answers

Answer:

E) 125 boxes

F) 800 candy bars

Step-by-step explanation:

E)

3,125/1.25 = 2500

2500/20 = 125

F)

1000/1.25 = 800

I believe it’s 40 but I may be wrong

Find the average (mean) of the given set of data.

22,15,31,40,27

Answers

Answer:

27

Step-by-step explanation:

Hi there!

We are given this set of data:

22, 15, 31, 40, 27

And we want to find the average (or the mean) of the set

To find the mean, we add up all of the values of the data set and then divide it by how many pieces of data we have

So first, add 22, 15, 31, 40 and 27 together

22+15+31+40+27=135

We have 5 pieces of data, so divide 135 by 5

135/5=27

The mean is 27

Hope this helps!

[tex]\boxed{\large{\mathbf{\blue{ANSWER~:) }}}}[/tex]

Data=22,15,31,40,27no of data=5sum of data=22+15+31+40+27=135

we know that,

[tex]\boxed{\sf{mean=\dfrac{sum ~of~ data}{no~ of~ data } }}[/tex]

According to the question,

[tex]{\sf{mean=\dfrac{135}{5 } }}[/tex]

[tex]\sf{ mean=27 }[/tex]

Therefore,

the average (mean) of the given set of data.(22,15,31,40,27) is 27.

can somebody please help me with this (geometry)

Answers

angle 2 is also 35 since they are alternate interior angles

What's the maximum area you can get for a rectangle with two sides along the x and y axes, and the opposite vertex in the first quadrant along the line y = 20 – 4x?

Answers

Answer:

Remember that a triangle rectangle of length L and width W has an area:

A = W*L

In our rectangle, we have two sides along the x and y axes.

So one of the vertices of our triangle rectangle is the point (0, 0)

And the other vertex, is along the line:

y = -4x + 20

So, if the opposite vertex is at the point:

(x₁, y₁)

We can define the length as the difference between the x-values of each vertex.

L = (x₁ - 0) = x₁

And the width, similarly, as:

W = (y₁ - 0) = y₁

Such that the point (x₁, y₁) is a solution for the equation y = -4x + 20, then we have:

y₁ = -4x₁ + 20

Then we can rewrite the width as:

W = -4x₁ + 20

Now, we can write the area of our rectangle as:

A = (x₁)*(-4x₁ + 20)

A = -4*x₁^2 + 20*x₁

Now we want to maximize the area, notice that the area is given by a quadratic equation with a negative leading coefficient.

Thus, the maximum will be at the vertex of that quadratic equation.

Remember that for a general quadratic equation:

y = a*x^2 + b*x + c

The x-value of the vertex is:

x = -b/(2*a)

so, in our case, the x-value of the vertex will be:

x₁ = -20/(-4*2) = 20/8 = 5/2

Now we can evaluate this in our area equation:

A = -4*(5/2)^2 + 20*(5/2)  = 49.36

This is the maximum area of the rectangle.

need help with this problem

Answers

Answer:

OA⊥OC

∴∠AOB+BOC= 90°

5x+8+6n-6=90

11x+2=90

x=90-2/11

x=88/11

x=8

∠BOC=6x-6

=6(8)-6

=48-6

∠BOC=42°

------------------------------

Hope it helps...

Have a great day!!

Answer:

[tex]<BOC = 42[/tex]

Step-by-step explanation:

In this problem, it is given that lines (AO) and (OC) are perpendicular. This means that when the lines intersect, the four angles formed each have a measure of (90) degrees. As per the given diagram, angles (<AOR) and (<ROC) add up to make (<AOC). Thus, one can form the following equation and solve for (x).

[tex](<AOR) + (<ROC) = (<AOC)[/tex]

Substitute,

[tex](6x-6)+(5x+8)=90[/tex]

Simplify,

[tex]11x+2=90[/tex]

Inverse operations,

[tex]11x+2=90[/tex]

[tex]11x=88\\\\x=8[/tex]

Substitute this into the equation for the measure of angle (<BOC) to find the numerical angle measure:

[tex]<BOC = 6x-6\\x = 8\\\\= 6(8)-6\\\\=48-6\\\\=42[/tex]

∛3x+7=∛2x+1
solve it please

Answers

Answer:

x = -6

Step-by-step explanation:

cube each side :

3x + 7 = 2x + 1

solve for x:

x = -6

The height of a rocket a given number of seconds after it is released is modeled by h (t) = negative 16 t squared + 32 t + 10. What does t represent? the number of seconds after the rocket is released the initial height of the rocket the initial velocity of the rocket the height of the rocket after t seconds The function V(r) = four-thirds pi r cubed can be used to find the volume of air inside a basketball given its radius. What does V(r) represent?

Answers

Answers:

t is the number of seconds after the rocket is released

V(r) is the volume of the ball with radius r.

====================================================

Explanation:

There isn't much to say in terms of explanation. These variables are simply definitions.

Determine the number of real solutions of -2x^2+ 5x – 3 = 0.
2
1 (double root)
5
0

Answers

Answer:

[tex]{ \tt{ - 2 {x}^{2} + 5x - 3 = 0 }} \\ { \tt{2 {x}^{2} - 5x + 3 = 0}} \\ { \tt{(2x - 3)(x - 1) = 0}} \\ { \tt{x = \frac{3}{2} \: and \: 1}}[/tex]

Answer:

2 i think

Step-by-step explanation:

sorry if im wrong

out of 500 bulbs, 0.2 are defective. how many are defctive

Answers

Answer:

100

Step-by-step explanation:

0.2*500=100

100 bulbs 500•0.2=100

A hundred chickadees can eat 100 kg of seeds in 100 days. How many kg of seeds can 10 chickadees eat in 10 days?

Answers

Answer:

1 kg

Step-by-step explanation:

Number of chickadees = 100

Quantity of seed eaten = 100 kg

Number of days = 100

Quantity of seeds each chickadee eats per day =Number of chickadees ÷ Quantity of seed eaten ÷ Number of days

= 100 ÷ 100 ÷ 100

= 1 ÷ 100

= 0.01 kg of seed

How many kg of seeds can 10 chickadees eat in 10 days?

= Quantity of seeds each chickadee eats per day × number of chickadee × number of days

= 0.01 kg × 10 × 10

= 1 kg

10 chickadees eat 1 kg of seeds in 10 days

Can someone help me with this math homework please!

Answers

Common difference = 2nd term - first term = -22.5-(-18)= -22.5+18 = -4.5

PLEASE HELP I BEG ASAP UWILL GET MARKED BRANLIEST

Answers

Answer:

3 C

4 B

5 B

6C

Step-by-step explanation:

3. the negative of the functiin reflects the original function on the x axis

4. multiplying by a number greater than 1 increases the slop of the original function

5. subtracting from x translates the function to the right

6. subtracting from f(x) translates the function down

Dwight wants to build a new rectangular beet farm, but he is first making a list of items he needs to purchase from the store. Dwight decides he will get 160 feet of
fencing to go around the farm that will have a length of 50 feet.
a. Find the width of the farm that Dwight wants to make. Include units.
b. Dwight decides that he will plant 1 beet seed for every square foot of land in this farm and he does not want to purchase any extra seeds. Exactly how many beet seeds should he purchase? Include units.

Answers

Answer:welcome

Step-by-step explanation: sorry I jsit really needed the points

WILL GIVE BRANLIEST AND BE SOOO HAPPY PLEASE HELP!!! 30 POINTS SHARE YOUR SMARTNESS!!

Answers

Answer:

s4 = 270

18+36+72+144 = 270

-- convergent sums to a single value

Step-by-step explanation:

Answer:

S4 = 270

Step-by-step explanation:

The 4th partial sum of the sequence is the sum of the first 4 terms:

S4 = a(1) + a(2) + a(3) + a(4)

Because the sequence is geometric any term can be written in terms of the previous one:

a(n) = r a(n - 1)

And more importantly in terms of the first term a(1) so that

a(2) = r a(1)

a(3) = r a(2) = r^2 a(1)

a(4) = r a (3) = r^3 a(1)

Then the 4th partial sum is

S4 = a(1) + r a(1) + r^2 a(1) + r^3 a(1)

S4 = a(1) (1 + r + r^2 + r^3)

With a a(1) = 18 and r = 2 we have

S4 = 18 (1 + 2 + 4 + 8) = 270

Therefore, the 4th partial sum is 270

Hello I’m trying to solve this problem I just don’t know how to do it.

BUT if someone could help and show me step by step that would be great I’m not trying to ask for the answer

I’m just trying learn how to solve it and so step by step there no rush Thank you

Answers

Step-by-step explanation:

To evaluate the proposed, the comprehension of linear data is required,

Slope: The rise/run or the accumulative unit distance between two differentiated points on a linear.

X-intercept: The peculiar point in which the observed linear data intersects the x-axis.

Y-intercept: The peculiar point in which the observed linear data intersects the y-axis.

1. To solve the following systems, first convert the Slope-Intercept formatting to Standard (General) form:

y = -5/3x + 3

3(Y = -5/3x + 3) Product by the denominator to eliminate the fraction.

3y = -5x + 9. Add 5x to the other expression as in standard form, the slope must be positive.

5x + 3y = 9 <== Standard (General) Form.

Y = 1/3x - 3

3(y = 1/3x - 3). Product by the denominator to eliminate the fraction.

3y = x - 9. Subtract by x to place the slope within the other expression of the equation.

-1(-x + 3y = -9). Now, product by -1 to contribute to a positive slope.

X - 3y = 9 <=== Standard (General) Form.

2. To solve for the x and y values, utilize the system of substitution:

1(5x + 3y = 9)Multiply equations by opposite slope to the other, and a positive to other.

-5(X - 3y = 9)

Evaluate,

+ 5x + 3y = 9. Now, add the systems.

-5x + 15y = -45

——————————

18y = -36

Y = -2

Thus, now that y is equated to -2, substitute that to either equation.

X - 3y = 9

X - 3(-2) = 9

X + 6 = 9

X = 3

Thus, x = 3, y = -2. This is their intersection point.

To plot these lines on the graph, execute the following,

Y = -5/3 x + 3

Start with the y-intercept. Draw a point on number 3 on the y-axis (vertical).

Starting with that point, go down 5 units, right 3 units.

* Remember, if there is a negative rise, go down. Positive, go up. If there is a negative run, go left. Positive, go right.

* Keep going 5 units down, right 3 units, until the graph allows.

2. Y = 1/3 x - 3

Similarly, conduct the same steps:

Starting with the y-intercept, draw a point on -3 on the vertical, or y-axis.

Beginning on that point, go up 1 unit, right 3 units.

Keep going up 1 units and 3 units right until the graph permits.

*I hope this helps.

Other Questions
Select all sets in which the number - 1/7 is an element. A.integersB.rational numbersC.whole numbersD.natural numbersE.irrational numbersF.real numbers How are gymnosperms in category gnetophyta usually pollinated?A. by windB. by handC. by insectsD. by water Geometry, please answer question ASAP URGENTPLEASE HELP ME!!There are many types of fitness resources available. These include online resources like fitness websites and blogs. Answer the following questions about fitness resources.Look online to find some fitness resources that could be used by teenagers. List 3 to 5 of the resources that you found, including the website/URL.Of the fitness resources you found, which is your favorite? Explain why you chose this resource as your favorite.Which fitness resource that you found is your least favorite? Explain why this resource is your least favorite. if f(x)= *picture* what is f(|a|) ? Read the following passage from Susan B. Anthony by Alma Lutz:If Sally Ann knows more about weaving than Elijah, reasoned eleven-year-old Susan with her father, then why dont you make her overseer?It would never do, replied Daniel Anthony as a matter of course. It would never do to have a woman overseer in the mill. (Pg. 1)Which character trait of Daniel Anthony is best illustrated in this passage?1. kind2. independent3. realistic4. forward thinkingPlease answer, It's for summer school, take your time if needed. Whatis the value of In e^4 why Malaysia has many rives? You observe a cell in a stained section of connective tissue. The cell has an indented nucleus and obvious cytoplasmic granules. Upon further testing, you determine that the granules contain histamine. This cell is most likely a(n) _____. Multiple choice question. mature neutrophil mast cell eosinophil B cell Are the triangle below similar what is communication? Enviro Company issues 10.50%, 10-year bonds with a par value of $430,000 and semiannual interest payments. On the issue date, the annual market rate for these bonds is 7.50%, which implies a selling price of 127.875. The straight-line method is used to allocate interest expense. 1. Using the implied selling price of 127.875. what are the issuers cash proceeds from issuance of these bonds? 2. What total amount of bond interest expense will be recognized over the life of these bonds? 3. What is the amount of bond interest expense recorded on the first interest payment date? Currently, your masks sell for $1 each. At that price you are able to sell 6000masks per week: A recent survey you conducted found that for every 10 centincrease in the price of the mask package you would sell 150 less mask packages.. Can someone plz help me ASAP!!!! What is 24/9 in simplest form It shows a lot in winter in America.Can someone change this sentence into Negative Form? find the area of the right triangle The mole fraction of NaCl in anaqueous solution is 0.132. Howmany moles of NaCl are present in1 mole of this solution?Molar MassNaCl: 58.44 g/molH2O: 18.016 g/mol NOMBRE DEL PUEBLO EN EL QUE HABLAN CON LA LENGUA CHACHIAYUDA PLIS Flag questionConsider the pressure and force acting on thedam retaining a reservoir of water. Suppose thedam is 500-m wideand the water is 80.0-mdeep at the dam, as illustrated below. What isthe average pressure on the dam due to thewater?